¿Por qué el impulso no es una función de la posición en la mecánica cuántica?

En mecánica cuántica, el operador de traducción de tiempo unitario tu ^ ( t 1 , t 2 ) es definido por tu ^ ( t 1 , t 2 ) | ψ ( t 1 ) = | ψ ( t 2 ) , y el operador hamiltoniano H ^ ( t ) se define como el límite de I tu ^ ( t , t + Δ t ) 1 Δ t como Δ t va a 0 . De manera similar, el operador de traducción espacial unidimensional está definido por T ^ ( X 1 , X 2 ) | X 1 = | X 2 y el operador de cantidad de movimiento pags ^ se define como el límite de I T ^ ( X , X + Δ X ) 1 Δ X como Δ X va a 0 . Mi pregunta es, ¿por qué el operador hamiltoniano puede ser una función del parámetro de tiempo? t , pero el operador de cantidad de movimiento no puede ser una función del parámetro de posición X ?

La única buena respuesta que obtuve a esta pregunta es que el tiempo no es un operador en la mecánica cuántica no relativista, mientras que la posición es un operador, por lo que el momento es una función de la posición y estropearía la relación de conmutación entre el momento y la posición. Pero esta explicación no tiene sentido para mí, porque considere el caso del momento angular de espín. Si R ^ z ( θ 1 , θ 2 ) denota el operador de rotación para rotaciones intrínsecas sobre el eje z (a diferencia de las rotaciones orbitales), luego el operador de momento angular de giro j ^ z (a diferencia de Beyoncé) se define como el límite de I R ^ z ( θ , θ + Δ θ ) 1 Δ θ como Δ θ va a 0 . Y todavía j ^ z no es función del ángulo θ , aunque no hay operador en la mecánica cuántica correspondiente a θ . (Hay otro operador llamado θ ^ , que es uno de los operadores de posición en coordenadas esféricas, pero que no tiene nada que ver con el espín y el θ de lo que estoy hablando; está relacionado con el momento angular orbital). Entonces, "el parámetro tiene un operador correspondiente" no parece la explicación correcta, ya que no explica por qué el momento angular de giro no puede ser una función del ángulo.

Tenga en cuenta que no estoy buscando una explicación ad hoc como "eso no tendría sentido físico en términos de cómo funcionan clásicamente la energía y el impulso". Quiero una explicación de los primeros principios de la mecánica cuántica.

Esta pregunta contiene muchos conceptos erróneos. El operador de traducción se define como T ( a ) | X = | X + a , donde a R . El operador de cantidad de movimiento se define como T ( 0 ) , por lo que es manifiestamente independiente de X : ni siquiera antes de tomar a 0 hizo T depender de X . La posición no es un parámetro, es el valor propio de un operador (y por lo tanto no tiene sentido afirmar que un segundo operador puede o no depender de él). El momento tampoco depende de la posición en la mecánica clásica, por lo que no está claro por qué debería hacerlo en QM.
De manera similar, el operador de rotación se define como R ^ ( α ) | X = | R ( α ) X , donde R ( α ) denota una matriz de rotación alrededor α ^ con ángulo | α | . El operador de momento angular se define como R ^ ( 0 ) , por lo que es manifiestamente independiente de θ : ni siquiera antes de tomar α 0 ¿Dependió de θ . Además, hay un operador que se puede identificar con θ ^ , y la pareja j ^ I , θ ^ j se comporta de manera muy similar a pags ^ I , X ^ j (aunque la imagen es más sutil).
@AccidentalFourierTransform Pero la pregunta es, ¿por qué el operador del traductor espacial no depende de X , mientras que el operador de traducción de tiempo puede depender de t ? Y la respuesta no puede ser porque la posición es un operador en la mecánica cuántica no relativista mientras que el tiempo no lo es, porque no explica por qué el operador de rotación intrínseco no depende de θ , Dado que θ tampoco es un operador.

Respuestas (7)

Creo que la respuesta se reduce a la causalidad. El problema típico que abordamos en la mecánica cuántica no relativista es "Dada una condición inicial ψ ( X , t = 0 ) , cual es la funcion de onda ψ ( X , T ) en otro momento T ?" El hecho de que el hamiltoniano H se permite que la generación de la traducción/evolución del tiempo dependa explícitamente del tiempo corresponde al hecho de que nosotros, los experimentadores, somos libres de controlar externamente el sistema de la manera que queramos, y el control externo no puede ser "predecido" endógenamente dentro del sistema. Expresado de otra manera, las influencias causales del impulso externo se propagan en el tiempo para afectar la función de onda futura.

Pero si el operador de cantidad de movimiento PAGS que genera traslaciones si se les permitiera depender no trivialmente del espacio, entonces por analogía con la ecuación de Schrödinger (y simplificar a una dimensión espacial) podríamos plantear un problema con una "condición inicial" ψ ( X = 0 , t ) y considere el problema de "evolucionar en el espacio" la función de onda en X según la ecuación diferencial I ψ / X = PAGS ( X ) ψ ( X ) . Si el experimentador fuera libre de cambiar externamente PAGS ( X ) , entonces las influencias de ese cambio necesitarían propagarse en una dirección similar al espacio para afectar la función de onda al mismo tiempo. t pero mas grande X , violando la causalidad.

Por tanto, el hecho de que el PAGS El operador debe ser "independiente del espacio", mientras que el H operador puede depender explícitamente del tiempo es un reflejo no relativista del hecho de que la teoría cuántica completa es relativista y que las influencias causales solo pueden propagarse en direcciones temporales. En un universo completamente no relativista, el operador de cantidad de movimiento PAGS probablemente podría depender lógicamente explícitamente de la posición, al igual que los bosones y los fermiones lógicamente podrían tener giros, pero en el mundo real "heredan" la relación estadística de giro de la teoría relativista subyacente.

Que hermosa respuesta!!!! Literalmente me detuve con asombro cuando estaba leyendo "Pero si se permitiera que el operador de impulso P que genera traslaciones dependiera de manera no trivial del espacio..."
Pero entonces, ¿por qué el operador de momento angular de giro no puede j ^ z depende del ángulo θ , el parámetro del operador de rotación intrínseco R ^ z ( θ ) ?
@KeshavSrinivasan Nuevamente, es lógica y matemáticamente posible, pero tal dependencia rompería explícitamente las simetrías que se supone que son válidas para la teoría relativista completa. Solo como explícito t dependencia en el hamiltoniano rompe la simetría traslacional del tiempo, una θ dependencia por la j El operador rompería la invariancia rotacional del espacio: habría una "dirección especial" y la física podría, por ejemplo, comportarse de manera diferente para los sistemas que miran en esa dirección frente a los que miran perpendicularmente a esa dirección. Es completamente lógicamente posible, aunque poco elegante,...
en un universo completamente no relativista y no se puede descartar de los primeros principios. Pero dado que la relatividad especial mezcla rotaciones y impulsos, tal dependencia arruinaría por completo toda la simetría de Poincaré de la teoría relativista. Si el hamiltoniano describiera un subsistema bajo el control de un experimentador, incluso podría hacer arreglos para simular un no trivial θ dependencia, por ejemplo, agregando manualmente energía al sistema cada vez que gira hacia el norte o algo así. Pero es técnicamente difícil y poco natural construir una teoría relativista "fundamental" que no sea...
... Lorentz invariante, por lo que en la práctica muchas teorías no relativistas heredan muchas simetrías del modelo estándar subyacente completo. Nuevamente, considere la analogía con los espines: es imposible derivar de los postulados de QM no relativista que espín-0 = bosón y espín-1 = fermión; uno podría definir fácilmente un espacio de Hilbert donde lo contrario es cierto. Pero hacerlo no es útil en el mundo real, debido a la teoría relativista subyacente.
QM no relativista es una teoría enormemente amplia y flexible, ¡y hay muy poco que podamos gobernar puramente a partir de los primeros principios! En la práctica, con el fin de hacer algún progreso por lo general necesitamos asumir algunos resultados básicos motivados por la experimentación o por alguna teoría más específica.
Pero hay situaciones en las que el hamiltoniano de una partícula depende del tiempo, pero eso no significa que se viole la simetría de traslación temporal del universo en su conjunto. Entonces, no hay razón por la cual el momento angular de una partícula que depende del ángulo viole la isotropía espacial del universo.
@KeshavSrinivasan En el mundo real, donde todas las interacciones fundamentales en el modelo estándar son independientes del tiempo, cualquier sistema descrito por un hamiltoniano dependiente del tiempo debe estar acoplado a algún "entorno" fuera del sistema mismo que genera la dependencia del tiempo, y que no está descrito por el hamiltoniano (o de lo contrario sería parte del sistema). De manera similar, estoy de acuerdo en que una partícula específica en una situación específica puede ser descrita por un hamiltoniano que rompe la invariancia rotacional, pero solo si la partícula está acoplada a algo externo que no está descrito por el ...
... hamiltoniano. El entorno es lo que rompe la simetría que subyace en el Modelo Estándar. Pero tal hamiltoniano no contaría como lo que yo llamo una "teoría fundamental". ¡Una "teoría fundamental" debe abarcar todo , incluidos los posibles entornos externos, experimentadores, etc.! Este tipo de teoría debe tener invariancia completa de Poincaré. Entonces sí, estoy de acuerdo, un sistema acoplado a algún entorno puede tener explícita θ dependencia, si el entorno rompe la simetría rotacional, como en el ejemplo que di arriba.
Bien, pero no estoy preguntando sobre una situación en la que el operador hamiltoniano de una partícula depende del ángulo, estoy hablando de una situación en la que el operador de momento angular de espín depende del ángulo.
El operador de momento angular de giro exacto se deriva en QFT relativista de la teoría de representación del grupo de Lorentz y, por lo tanto, hereda la simetría angular de ese grupo. Pero en muchas situaciones de materia condensada, puede definir un operador de momento angular de giro efectivo que, de hecho, depende del ángulo. Esto se llama división del campo cristalino .
Muy claro y elocuente; esta debería ser la respuesta aceptada. @KeshavSrinivasan, creo que la respuesta a su objeción es la siguiente. Si el único hamiltoniano que alguien estudió fue el hamiltoniano donde se toma todo el universo como el sistema bajo estudio, entonces (1) este hamiltoniano sería independiente del tiempo y (2) la traducción del tiempo funcionaría igual que la traducción espacial, por lo que su la pregunta se desvanecería. Pero dado que nos gusta estudiar sistemas que son más pequeños que todo el universo, donde un experimentador puede controlar el entorno, entonces tratamos el tiempo de manera diferente.

Su idea de lo que se define en términos de lo que es un poco engañosa. Usualmente, el físico toma los generadores infinitesimales H , pags , X como operadores autoadjuntos dados y define las transformaciones finitas a ser tu ( t ) = Exp ( I t H ) , T ( ξ ) = Exp ( I ξ pags ) , S ( π ) = Exp ( I π X ) siguiendo el teorema de Stone . Si H depende del tiempo entonces tu ( t ) se convierte en la serie Dyson para tu ( t , t 0 ) en la imagen de interacción.

En cuanto a la definición del operador de cantidad de movimiento en sí: simplemente se define como el operador con [ X , pags ] = I . Por el teorema de Stone-von Neumann , todas las formas posibles de realizar operadores con esa relación de conmutación son esencialmente las mismas que en L 2 ( R ) , donde X es la multiplicación por la variable y pags es la diferenciación. Las relaciones de conmutación también codifican que la transformación T ( ξ ) actúa como una traducción sobre la posición y que S ( π ) actúa como una traducción del impulso, vea también esta respuesta mía . Pero crucialmente, pags es por definición un único operador fijo . Simplemente no está permitido depender de nada.

Finalmente, su confusión parece surgir básicamente de escribir todas esas transformaciones con dos parámetros, es decir tu ( t 0 , t 1 ) , T ( X 1 , X 2 ) . Solo se permite que la evolución temporal dependa de dos parámetros de esa manera, y solo en el caso de un hamiltoniano dependiente del tiempo. Todas las demás transformaciones son grupos de un parámetro como en el teorema de Stone, generados por un solo operador autoadjunto. Esto no se muestra, pero se asume . Suponemos que el operador de rotación R ( θ 1 , θ 2 ) realmente solo le importa la diferencia entre los dos ángulos, es decir, en realidad es solo una función R ( θ 1 θ 2 ) y suponemos que la traducción T ( X 1 , X 2 ) es realmente justo T ( X 1 X 2 ) . Podrías suponer de otra manera, pero eso no es lo que hacemos en la mecánica cuántica estándar.

Suponemos que para todas esas transformaciones porque queremos que la T ( X 1 , X 2 ) ser en realidad una representación (unitaria) del grupo de traducción R , y el R ( θ 1 , θ 2 ) ser una representación del grupo de rotación S O ( 3 ) . Y esos grupos no contienen las transformaciones "girar desde el ángulo θ 1 para θ 2 ", pero "rotar por ángulo θ ", por lo que el operador también solo dependerá de la diferencia, y no de los puntos de inicio/fin de la transformación.

El caso de la evolución del tiempo es diferente, aunque se podría decir que hay un "grupo de traducción del tiempo", lo que realmente queremos es un operador que codifique la evolución de un sistema dinámico. Y en un sistema dinámico podemos imaginar fácilmente que en algún momento t 0 algo se "encende/apaga" que altera la dinámica del sistema después de ese punto, de modo que tu ( t 1 , t 2 ) es diferente dependiendo de si ambos t 1 , t 2 son antes o despues t 0 .

no estoy de acuerdo Es perfectamente estándar tomar los operadores unitarios como primitivos y definir los generadores infinitesimales en términos de ellos. Eso es lo que hacen innumerables libros de texto, incluidos Townsend y (si mal no recuerdo) Sakurai. Y luego la relación de conmutación se deriva, no se asume. Y de manera similar, la serie Dyson también se deriva, no se asume. Es en el contexto de ese enfoque que hago mi pregunta.
@KeshavSrinivasan Entonces mi último párrafo aún se aplica: que el operador de traducción T ( X 1 , X 2 ) depende en realidad solo de X 1 X 2 y es, por lo tanto, un subgrupo de un parámetro al que podemos aplicar el teorema de Stone para obtener un solo operador de momento independiente es una suposición .
Bueno, esa es precisamente mi pregunta: ¿por qué asumimos que T ( X 1 , X 2 ) depende solo de X 1 X 2 , y R z ( θ 1 , θ 2 ) depende solo de θ 1 θ 2 , pero permitimos tu ( t 1 , t 2 ) depender directamente de t 1 y t 2 ?
@KeshavSrinivasan Agregué un razonamiento para eso.
Bueno, eso simplemente lleva el problema un paso atrás: ¿por qué permitimos que las cosas sean diferentes en diferentes puntos en el tiempo, pero no diferentes de manera similar en diferentes puntos en el espacio o en diferentes direcciones?
@KeshavSrinivasan ¿Porque esta forma de describir los sistemas físicos funciona? No estoy seguro de qué tipo de razón está buscando realmente: al final, las suposiciones hechas en nuestras teorías físicas están ahí porque la teoría resultante predice correctamente los resultados de los experimentos. Además, ingenuamente, ¿no estaría de acuerdo en que poner algo en posición X 1 para X 1 + B y algo en X 2 para X 2 + B debería ser la misma operación sin importar qué X 1 y X 2 ¿están?
Sí, pero también ingenuamente creería lo mismo sobre t 1 y t 2 . Ahora alguien podría corregir mi suposición ingenua y decirme que las cosas pueden variar con el tiempo de una manera que contradiría mi suposición sobre t 1 y t 2 , pero le preguntaría a esa persona por qué las cosas no pueden variar de manera similar con la posición de una manera que contradiría mi suposición ingenua sobre X 1 y X 2 , o variar con el ángulo de una manera que contradiría mi suposición ingenua sobre θ 1 y θ 2 .
En cualquier caso, esto es lo que realmente busco: no creo que tengas razón al decir que es solo un postulado fundamental que hacemos en base a la observación experimental. Creo que es probablemente una consecuencia de algo más, y quiero encontrar qué es ese algo. (Y no creo que ese algo sea la relación de conmutación, porque eso no explicaría el caso del momento angular de espín).
@KeshavSrinivasan Puede darse el caso de que ciertos aspectos de los modelos cuánticos se puedan describir como "consecuencias de otra cosa", pero ¿por qué importa eso? El nivel en el que ACuriousMind ha abordado esta discusión está bastante abajo en la cadena de "por qué" en lo que respecta a los puntos de partida y hace que la mayoría de las elecciones que se hacen comúnmente en ciertos modelos de QM sean bastante plausibles. Entonces, comenzamos con este modelo plausible, hacemos predicciones y funcionan bastante bien. Tengo curiosidad por saber qué forma de respuesta buscas si no es algo como este.

Creo que la noción de que el operador de posición o de momento es una función del otro está un poco mal definida. Me doy cuenta de que no quiere explicaciones de la física clásica, así que disculpe por el momento la analogía con la mecánica hamiltoniana.

En la mecánica hamiltoniana tratamos con un 2 norte -espacio dimensional, en el que debería haber una forma bidimensional, es decir, un tensor bidimensional antisimétrico ω I j , que debe ser no degenerado, ω I j v j 0 a no ser que v j = 0 , y cerrado, [ I ω j k ] = 0 donde los corchetes significan antisimetrización. Todas estas condiciones son independientes de las coordenadas. Ya que ω I j no es degenerado, tiene inversa ω I j . Si F , gramo son funciones en este 2 norte -espacio dimensional, podemos definir una operación llamada corchete de Poisson de F y gramo por

{ F , gramo } = ω I j ( ω I k k F ) ( ω j yo yo gramo ) .
Tenga en cuenta que el corchete de Poisson se define de forma independiente de las coordenadas. La otra cosa que necesitamos hacer mecánica hamiltoniana es una función hamiltoniana. H , que define la dinámica a través
F ˙ = { F , H } .

Los componentes de la forma 2 ω I j son, por supuesto, dependientes de las coordenadas. Ahora, por un teorema de Darboux, siempre es posible (localmente) encontrar lo que se llama coordenadas canónicas X I , I = 1 , , 2 norte tal que ω I j toma la siguiente forma

ω I j = [ 0 I norte I norte 0 ]
donde I norte es el norte × norte matriz de identidad. Dejar q I = X I , I = 1 , , norte ; pags I = X I + norte , I = 1 , , norte . Entonces puedes resolver eso
{ F , gramo } = F q I gramo pags I F pags I gramo q I .
En particular,
{ q I , pags j } = d I j { q I , q j } = { pags I , pags j } = 0
y puedo recuperar fácilmente las ecuaciones de Hamilton en la forma canónica.

En un sistema de coordenadas particular como el descrito, un sistema de coordenadas canónico, el q I se denominan las posiciones o coordenadas, y las pags I se denominan momentos. Ninguno es función del otro. Pero eso es cierto para cualquier sistema de coordenadas: todas las coordenadas son mutuamente independientes. Uno podría igualmente usar v I = pags I mi A I ( q j ) como la segunda mitad de las coordenadas. Los componentes de la forma 2 serán más complicados y la fórmula para el soporte de Poisson no será tan agradable, pero ( X I , v I ) es un sistema de coordenadas perfectamente fino.

Ahora pasemos a la mecánica cuántica. En mecánica cuántica tenemos operadores en lugar de conmutadores en lugar de corchetes de Poisson. La dinámica está dada por

O ˙ ^ = I [ O ^ , H ^ ]
y por analogía con la mecánica hamiltoniana introducimos como operadores observables X ^ I , pags ^ I que satisfacen
[ X ^ I , pags ^ j ] = I d I j [ X ^ I , X ^ j ] = [ pags ^ I , pags ^ j ] = 0.
Decimos que estos operadores forman la base de un álgebra de Lie. Pero así como soy libre de cambiar las coordenadas en el caso clásico, soy libre de cambiar la base en el caso cuántico. No tiene sentido decir que el operador de posición es una función del operador de cantidad de movimiento o viceversa porque todos son elementos básicos independientes entre sí en el álgebra de Lie.

El caso cuántico también se puede formular directamente en términos de un cambio de coordenadas. Luego, en lugar del corchete de Poisson en 2 norte espacio de fase -dimensional, uno tiene el corchete de Moyal . Al igual que el corchete de Poisson, el corchete de Moyal actúa como un operador diferencial y su expresión es particularmente simple en sistemas de coordenadas especiales, pero no es necesario usar tales coordenadas.

Pero esto no explica por qué el operador de momento angular de espín no es una función del ángulo θ que el operador de rotación intrínseco R ^ z es una función de. θ no tiene ningún operador u observable que le corresponda, por lo que es como t en ese sentido. Y sin embargo, el operador hamiltoniano H se permite que sea una función de t .
@KeshavSrinivasan ¿Por qué dice que no hay operador para θ y que no es un observable? Es arccos ( z ^ / r ^ ) ¿No es suficientemente bueno para ti? ¿Crees que la mecánica cuántica no puede describir cuerpos rígidos? (¿Cómo entonces, puede la mecánica clásica?)
eso es diferente θ , como dejé claro en mi pregunta. Que θ , que es uno de los operadores de posición en coordenadas esféricas, es un parámetro del operador de rotación orbital. Mientras que el θ Estoy hablando de un parámetro del operador de rotación intrínseco, es decir, el operador de rotación conectado al giro. El operador de momento angular orbital tiene una relación de conmutación con el θ que estás hablando. Pero no hay un operador de ángulo similar con el que el operador de momento angular de espín tenga una relación de conmutación.
Entonces, ¿por qué existiría tal operador? En la medida en que existe una teoría clásica del espín, es decir, del momento angular intrínseco, no es una teoría de un rotor rígido. Consulte, por ejemplo, aquí physics.stackexchange.com/questions/108387/…
Sí, ese es precisamente mi punto, tal θ El operador no existe. Entonces, ¿por qué no puedo j ^ z ser una función de θ , Tal como H ^ es una función de t ?

En realidad, hay escenarios en los que el operador de impulso podría depender de la posición. Considere, por ejemplo, la propagación de la luz a través de un medio aleatorio. Si incorporamos el efecto de este medio aleatorio en la evolución unitaria del campo a través del medio, entonces el operador de cantidad de movimiento del que se derivaría dependería de la posición debido a la aleatoriedad del medio.

Cuando el operador de cantidad de movimiento no depende de la posición, refleja el hecho de que el sistema bajo investigación obedece a la invariancia de traslación espacial y, por lo tanto, apoya la conservación de la cantidad de movimiento. En un medio aleatorio, la cantidad de movimiento no se conserva, porque el medio podría causar la dispersión de la luz, lo que implica un cambio en la cantidad de movimiento.

Sin embargo, lo mismo se aplica para el hamiltoniano. Si el hamiltoniano tiene una dependencia temporal explícita, entonces el sistema no es invariante con respecto a las traslaciones temporales y entonces se rompe la conservación de la energía.

En el nivel fundamental, sabemos que tanto el impulso como la energía se conservan. Esto se refleja en el hecho de que ni el hamiltoniano ni el operador de impulso dependen explícitamente del tiempo o la posición. Por ejemplo, en las teorías cuánticas de campos (como QED) la dependencia de las coordenadas espacio-temporales está restringida a la de los campos y no aparece explícitamente en el Lagrangiano. La invariancia de traslación implícita en las coordenadas espacio-temporales conduce a una corriente de Noether, el tensor de energía-momento, del cual se obtienen las expresiones para el hamiltoniano y los operadores de momento, expresadas puramente en términos de los campos y sus derivados.

Como ejemplo más simple, ¿podemos considerar la partícula en un sistema de caja? Aquí, el operador de impulso solo se puede definir en un pequeño intervalo (por lo tanto, dependiente de la posición), aunque existen otros problemas relacionados con la autoadjunción del impulso, pero ese es otro tema de interés.

Primero piense en el hamiltoniano. El hamiltoniano es un generador de traslaciones de tiempo, por lo que puede ser una función de t . Puede ser dependiente o independiente del tiempo. Esto codifica cómo evoluciona el sistema en función del tiempo. Ahora, matemáticamente hablando, el impulso es el generador de traslaciones espaciales, por lo que, en principio, puede ser una función de un parámetro de posición . Entonces podría ser dependiente de la posición o independiente de la posición. Esto nos diría cómo evoluciona el sistema en el espacio .. La mayoría de los experimentadores no hacen experimentos moviendo sistemas cuánticos en el espacio, pero sí, hacen experimentos moviendo sistemas en el tiempo. Es concebible en el futuro, cuando las técnicas de control cuántico sean muy avanzadas y los sistemas cuánticos puedan protegerse de la decoherencia, que algún día un experimentador necesite conocer un operador de momento en función de un parámetro de posición. Recuerde también que esto es física no relativista, la posición y el tiempo no están en el mismo plano. No hay razón para pensar que deberían comportarse de la misma manera. Entonces, para responder a su pregunta directamente, el operador de impulso puede, en principio, ser una función de un parámetro de posición .

Estoy bastante seguro de que si el impulso fuera una función de la posición, sería fácilmente observable experimentalmente, ya que la relación de conmutación posición-impulso no se mantendría y, por lo tanto, observaría violaciones del principio de incertidumbre de Heisenberg, que simplemente no observamos.
La relación de conmutación de Heisenberg es verdadera si el operador de cantidad de movimiento es independiente de un parámetro de posición, pero eso no dice nada acerca de si es posible que la cantidad de movimiento dependa de un parámetro de posición. Bien podría haber una ecuación diferencial para resolver la cantidad de movimiento en algún sistema complicado y mientras el operador de cantidad de movimiento permaneciera hermético, el operador que actúa sobre el estado cuántico obtenido al exponenciar el operador de cantidad de movimiento permanecería unitario y, por lo tanto, un operador cuántico legítimo.
Esto debería verificarse, pero estoy bastante seguro de que cualquier hamiltoniano que combinara la posición con el impulso de una manera lo suficientemente complicada le daría un impulso en función de la posición.

¿Por qué el operador hamiltoniano puede ser una función del parámetro de tiempo t, pero el operador de cantidad de movimiento no puede ser una función del parámetro de posición x?

Un operador de cantidad de movimiento puede ser una función de X . Citaré extensamente de las páginas 57-58 de "Gauge Theories in Particle Physics, 2nd Ed" de Aitchison & Hey. así que esto es demasiado largo para ser un comentario:

El punto esencial es que (en una dimensión, digamos) pags ^ se define en última instancia por el conmutador ( = 1 )

[ X ^ , pags ^ ] = I

Ciertamente, la elección familiar

pags ^ = I X

satisface la relación de conmutación. Pero también podemos añadir cualquier función de X para pags ^ , y esto modificado pags ^ seguirá siendo satisfactoria ya que X conmuta con cualquier función de X . Consideraciones más detalladas de Dirac mostraron que esta función arbitraria en realidad debe tener la forma F X , donde F es arbitrario Por lo tanto

pags ^ = I X + F X

es un operador de cantidad de movimiento aceptable.

Puede haber diez mil operadores que satisfagan la relación de conmutación, pero presumiblemente solo hay un operador que es igual al límite de I T ^ ( X , X + Δ X ) 1 Δ X como Δ X va a 0. Me interesa saber por qué ese operador no puede ser una función de posición.
@KeshavSrinivasan, es justo, pero permítame señalar lo siguiente en su pregunta: "por lo que el impulso es una función de la posición y estropearía la relación de conmutación posición-impulso" .

Muchas respuestas matemáticas aquí. No estoy seguro de cuán satisfactorios se "sentirán" ya que los que he leído no parecen dar ninguna intuición de por qué el impulso no es el derivado de la posición.

Creo que la respuesta intuitiva es simplemente que la cantidad de movimiento de una onda no es lo mismo que la cantidad de movimiento de una partícula. Este operador de cantidad de movimiento ( I D D X ) es más como un "operador de impulso de onda" y mide el impulso del paquete de ondas asociado con el estado cuántico. Pero nada le impide realizar un seguimiento del "momento de la partícula" y calcular explícitamente metro D X D t .

Aquí hay una respuesta decente para desarrollar la intuición sobre cuál es el impulso de una onda:

Para aclarar el problema, consideremos un modelo simplificado de la cuerda: la cuerda se extiende a lo largo de la dirección x y está formada por masas conectadas por resortes. Para mayor claridad conceptual, suponga que estas masas solo se pueden mover hacia arriba y hacia abajo (a lo largo de y; esto podría imponerse en un modelo mecánico haciendo que las masas se deslicen hacia arriba y hacia abajo sobre pequeños cables). En ese caso, el momento mecánico está claramente solo en la dirección y (todo el movimiento es a lo largo de y), y no hay momento mecánico en la dirección de propagación de la onda (x). Dependiendo de la forma del paquete de ondas, el momento total en la dirección y también puede ser cero, si algunas masas se mueven hacia arriba mientras que otras se mueven hacia abajo.

Este operador de "momento de onda" resulta particularmente útil cuando se describe la energía del sistema (y también se puede usar para "generar" movimiento en posición aplicando el operador varias veces, pero este movimiento es el movimiento de todo el sistema). paquete de ondas).

Ahora, ¿por qué es la derivada de x específicamente? Podemos ganar algo de intuición mirando la solución de onda plana a la ecuación de Schroinger:

ψ ( X , t ) = mi I ( pags X mi t )

Tratemos de resolver para p y averiguar qué es p específicamente. Podemos hacer que p "baje" de la exponencial aplicando una derivada parcial.
donde esta p

ψ X = X mi I ( pags X mi t ) = I pags mi I ( pags X mi t )

pags ψ = I X ψ

Por lo tanto, podemos inferir qué es este operador p moviendo todos los términos para mostrar explícitamente qué debe estar haciendo p como operador:

pags ^ = I X